23科大应统

23科大专硕

一、填空题

  1. 投掷硬币 n n n 次, 已知正面出现了 k k k 次, 则前两次是正面的概率是 ‾ \underline{\qquad} .

Solution: k ( k − 1 ) n ( n − 1 ) \frac{k(k-1)}{n(n-1)} n(n1)k(k1).

P ( X 1 = 1 , X 2 = 1 ∣ ∑ i = 1 n X i = k ) = P ( X 1 = 1 , X 2 = 1 , ∑ i = 3 n X i = k − 2 ) C n k ( 1 2 ) n = 1 2 ⋅ 1 2 ⋅ C n − 2 k − 2 ( 1 2 ) n − 2 C n k ( 1 2 ) n = k ( k − 1 ) n ( n − 1 ) . \begin{aligned} P\left( X_1=1,X_2=1\left| \sum_{i=1}^n{X_i}=k \right. \right) &=\frac{P\left( X_1=1,X_2=1,\sum_{i=3}^n{X_i}=k-2 \right)}{C_{n}^{k}\left( \frac{1}{2} \right) ^n}\\ &=\frac{\frac{1}{2}\cdot \frac{1}{2}\cdot C_{n-2}^{k-2}\left( \frac{1}{2} \right) ^{n-2}}{C_{n}^{k}\left( \frac{1}{2} \right) ^n}=\frac{k\left( k-1 \right)}{n\left( n-1 \right)}.\\ \end{aligned} P(X1=1,X2=1 i=1nXi=k)=Cnk(21)nP(X1=1,X2=1,i=3nXi=k2)=Cnk(21)n2121Cn2k2(21)n2=n(n1)k(k1).

  1. 设有三角形 A B C ABC ABC, 某人最开始站在 A A A 点, 随机的向另外两个点走去, 随后每次如此, 问第 n n n 次他走向 A A A 点的概率是 ‾ \underline{\qquad} .

Solution: 1 3 ( 1 − ( − 1 2 ) n ) \frac{1}{3}\left( 1-\left( -\frac{1}{2} \right) ^n \right) 31(1(21)n).

考虑状态法, 设 a n , b n , c n a_n,b_n,c_n an,bn,cn 分别是它第 n n n 次之后位于三点的概率, 有 a 0 = 1 , b 0 = 0 , c 0 = 0 a_0=1,b_0=0,c_0=0 a0=1,b0=0,c0=0, 以及 a 1 = 0 , b 1 = 1 2 , c 1 = 1 2 a_1=0,b_1=\frac{1}{2},c_1=\frac{1}{2} a1=0,b1=21,c1=21. 显然所求概率应为 p n = 1 − a n − 1 2 p_n = \frac{1-a_{n-1}}{2} pn=21an1, 即它上一次之后不在 A A A 点的概率, 再等分给他可以前往的两点.

用全概率公式有
{ a n + 1 = 0 ⋅ a n + 1 2 ⋅ b n + 1 2 ⋅ c n , b n + 1 = 1 2 ⋅ a n + 0 ⋅ b n + 1 2 ⋅ c n , a n + 1 = 1 2 ⋅ a n + 1 2 ⋅ b n + 0 ⋅ c n , ⇒ ( a n + 1 b n + 1 c n + 1 ) = ( 0 1 2 1 2 1 2 0 1 2 1 2 1 2 0 ) ( a n b n c n ) , \begin{cases} a_{n+1}=0\cdot a_n+\frac{1}{2}\cdot b_n+\frac{1}{2}\cdot c_n,\\ b_{n+1}=\frac{1}{2}\cdot a_n+0\cdot b_n+\frac{1}{2}\cdot c_n,\\ a_{n+1}=\frac{1}{2}\cdot a_n+\frac{1}{2}\cdot b_n+0\cdot c_n,\\ \end{cases}\quad \Rightarrow \quad \left( \begin{array}{c} a_{n+1}\\ b_{n+1}\\ c_{n+1}\\ \end{array} \right) =\left( \begin{matrix} 0& \frac{1}{2}& \frac{1}{2}\\ \frac{1}{2}& 0& \frac{1}{2}\\ \frac{1}{2}& \frac{1}{2}& 0\\ \end{matrix} \right) \left( \begin{array}{c} a_n\\ b_n\\ c_n\\ \end{array} \right) , an+1=0an+21bn+21cn,bn+1=21an+0bn+21cn,an+1=21an+21bn+0cn, an+1bn+1cn+1 = 021212102121210 anbncn ,
可以用这个矩阵 n n n 次方去算, 该方法称为马尔科夫链. 但是根据对称性, 我们知道 b n = c n b_n=c_n bn=cn, 故 b n = c n = 1 − a n 2 b_n=c_n=\frac{1-a_n}{2} bn=cn=21an, 到最后只剩 a n a_n an 一个序列了, 我们反表示出 a n = 1 − 2 p n + 1 a_n=1-2p_{n+1} an=12pn+1, b n = c n = p n + 1 b_n=c_n=p_{n+1} bn=cn=pn+1, 代入第一个全概率公式即为
1 − 2 p n + 2 = p n + 1 , ⇒ p n + 1 − 1 3 = − 1 2 ( p n + 1 − 1 3 ) , 1-2p_{n+2}=p_{n+1},\quad \Rightarrow \quad p_{n+1}-\frac{1}{3}=-\frac{1}{2}\left( p_{n+1}-\frac{1}{3} \right) , 12pn+2=pn+1,pn+131=21(pn+131),
代入 p 0 = 0 p_0=0 p0=0, 解得
p n = 1 3 + ( − 1 2 ) n ( 0 − 1 3 ) = 1 3 ( 1 − ( − 1 2 ) n ) . p_n=\frac{1}{3}+\left( -\frac{1}{2} \right) ^n\left( 0-\frac{1}{3} \right) =\frac{1}{3}\left( 1-\left( -\frac{1}{2} \right) ^n \right) . pn=31+(21)n(031)=31(1(21)n).

  1. 已知将 A , B , C A,B,C A,B,C 三个子母输入信道, 输出正确的概率是 0.8 0.8 0.8, 输出为其他字母的概率是 0.1 , 0.1 0.1,0.1 0.1,0.1. 现在, 等概率地输入 A A A A , B B B B , C C C C AAAA,BBBB,CCCC AAAA,BBBB,CCCC, 且观测到 A C B A ACBA ACBA, 问输入是 A A A A AAAA AAAA 的概率为 ‾ \underline{\qquad} .

Solution: 0.8 0.8 0.8.

利用全概率公式得
P ( o u t : A C B A ) = 1 3 ⋅ ( 0. 8 2 ⋅ 0. 1 2 ) + 1 3 ⋅ ( 0. 1 3 ⋅ 0.8 ) + 1 3 ⋅ ( 0. 1 3 ⋅ 0.8 ) = 0.00266667 , P\left( \mathrm{out}:ACBA \right) =\frac{1}{3}\cdot \left( 0.8^2\cdot 0.1^2 \right) +\frac{1}{3}\cdot \left( 0.1^3\cdot 0.8 \right) +\frac{1}{3}\cdot \left( 0.1^3\cdot 0.8 \right) =0.00266667, P(out:ACBA)=31(0.820.12)+31(0.130.8)+31(0.130.8)=0.00266667,
利用贝叶斯公式得
P ( i n : A A A A ) = 1 3 ⋅ ( 0. 8 2 ⋅ 0. 1 2 ) P ( o u t : A C B A ) = 0.8. P\left( \mathrm{in}:AAAA \right) =\frac{\frac{1}{3}\cdot \left( 0.8^2\cdot 0.1^2 \right)}{P\left( \mathrm{out}:ACBA \right)}=0.8. P(in:AAAA)=P(out:ACBA)31(0.820.12)=0.8.

  1. 检验的 p p p 值是否为统计量? ‾ \underline{\qquad} .

Solution: 是.

p p p 值依赖于观测到的样本, 属于统计量.

  1. 下列说法正确的个数是 ‾ \underline{\qquad} .
    (1) R 2 R^2 R2 越小说明方程的拟合越好;
    (2) R 2 R^2 R2 越大说明方程的拟合越好;
    (3) 残差 e = y ^ − y e=\hat{y}-y e=y^y 越大说明方程的拟合越好;
    (4) 残差分析图中, 点的分布越平稳说明方程的拟合越好, 且点分布带状图越窄, 说明拟合精度越高.

Solution: 2.

(1) (3) 显然错误, (2) (4) 正确.

  1. 对任意三角形 A B C ABC ABC 内部取一点 P P P, 在 B C BC BC 上取 Q Q Q, 则直线 P Q PQ PQ A B AB AB 相交的概率是 ‾ \underline{\qquad} .
    A. 1 2 \frac{1}{2} 21
    B. ∣ B C ∣ ∣ A B ∣ + ∣ B C ∣ \frac{|BC|}{|AB|+|BC|} AB+BCBC
    C. ∣ B C ∣ 2 ∣ A B ∣ + ∣ B C ∣ \frac{|BC|^2}{|AB|+|BC|} AB+BCBC2
    D. ∣ A B ∣ + ∣ A C ∣ + ∣ B C ∣ 2 ∣ A B ∣ + ∣ A C ∣ + ∣ B C ∣ \frac{|AB|+|AC|+\frac{|BC|}{2}}{|AB|+|AC|+|BC|} AB+AC+BCAB+AC+2BC 不确定

Solution: A.

设三角形的边 B C = a BC=a BC=a, B B B 为原点, B C BC BC x x x 轴, 则 Q ∼ U ( 0 , a ) Q\sim U(0,a) QU(0,a), P ∼ U ( Δ A B C ) P\sim U(\Delta ABC) PU(ΔABC). 先取定 Q = ( q , 0 ) Q=(q,0) Q=(q,0), 连接 A Q AQ AQ, P P P 要落在 Δ A B Q \Delta ABQ ΔABQ 里才能满足题设条件, 故有
P r ( P ∈ Δ A B Q ∣ Q = q ) = S Δ A B Q S Δ A B C = q a , \mathrm{Pr}\left( P\in \Delta ABQ|Q=q \right) =\frac{S_{\Delta ABQ}}{S_{\Delta ABC}}=\frac{q}{a}, Pr(PΔABQQ=q)=SΔABCSΔABQ=aq,
再让 q q q 动起来, 有
P r ( P Q ∩ A B ) = ∫ 0 a q a ⋅ 1 a d q = 1 2 . \mathrm{Pr}\left( PQ\cap AB \right) =\int_0^a{\frac{q}{a}\cdot \frac{1}{a}}dq=\frac{1}{2}. Pr(PQAB)=0aaqa1dq=21.

  1. X 1 , ⋯   , X 9 X_1,\cdots,X_9 X1,,X9 是 i.i.d. 的 N ( 0 , 1 ) N(0,1) N(0,1) 随机变量, 下列正确的是 ‾ \underline{\qquad} .
    A. X 1 2 + X 2 2 + X 3 2 X 4 2 + ⋯ + X 9 2 ∼ F ( 3 , 6 ) \frac{X_1^2+X_2^2+X_3^2}{X_4^2+\cdots+X_9^2}\sim F(3,6) X42++X92X12+X22+X32F(3,6)
    B. 2 X 1 2 + X 2 2 + X 3 2 X 4 2 + ⋯ + X 9 2 ∼ F ( 3 , 6 ) 2\frac{X_1^2+X_2^2+X_3^2}{X_4^2+\cdots+X_9^2}\sim F(3,6) 2X42++X92X12+X22+X32F(3,6)
    C. X 1 2 X 1 2 + X 2 2 ∼ F ( 1 , 2 ) \frac{X_1^2}{X_1^2+X_2^2} \sim F(1,2) X12+X22X12F(1,2)
    D. 2 X 1 2 X 1 2 + X 2 2 ∼ F ( 1 , 2 ) \frac{2X_1^2}{X_1^2+X_2^2} \sim F(1,2) X12+X222X12F(1,2)

Solution: B.

注意 C, D 并不满足分子分母的独立性.

  1. 已知 X ∼ P ( λ ) X\sim \mathcal{P}(\lambda) XP(λ), Y ∼ P ( μ ) Y\sim \mathcal{P}(\mu) YP(μ), 且它们独立, 求 E ( X ∣ X + Y = n ) = ‾ E(X|X+Y=n)=\underline{\qquad} E(XX+Y=n)=.

Solution: λ n λ + μ \frac{\lambda n}{\lambda+\mu} λ+μλn.

P ( X = k ∣ X + Y = n ) = P ( X = k , Y = n − k ) P ( X + Y = n ) = λ k k ! e − λ μ n − k ( n − k ) ! e − μ ( λ + μ ) n n ! e − ( λ + μ ) = C n k ( λ λ + μ ) k ( μ λ + μ ) n − k , P\left( X=k|X+Y=n \right) =\frac{P\left( X=k,Y=n-k \right)}{P\left( X+Y=n \right)}=\frac{\frac{\lambda ^k}{k!}e^{-\lambda}\frac{\mu ^{n-k}}{\left( n-k \right) !}e^{-\mu}}{\frac{\left( \lambda +\mu \right) ^n}{n!}e^{-\left( \lambda +\mu \right)}}=C_{n}^{k}\left( \frac{\lambda}{\lambda +\mu} \right) ^k\left( \frac{\mu}{\lambda +\mu} \right) ^{n-k}, P(X=kX+Y=n)=P(X+Y=n)P(X=k,Y=nk)=n!(λ+μ)ne(λ+μ)k!λkeλ(nk)!μnkeμ=Cnk(λ+μλ)k(λ+μμ)nk,

因此 X + Y = n X+Y=n X+Y=n 时, X X X 的条件分布是 B ( n , λ λ + μ ) B(n,\frac{\lambda}{\lambda +\mu}) B(n,λ+μλ), 故期望是 λ n λ + μ \frac{\lambda n}{\lambda+\mu} λ+μλn.

  1. CLT,忘了,比较简单

  2. 忘了

二、计算分析题

  1. (25分) 已知 X ∼ f ( x ) = 1 2 e − 1 2 x , x > 0 X\sim f(x)=\frac{1}{2}e^{-\frac{1}{2}x},x>0 Xf(x)=21e21x,x>0, Y ∼ U ( 0 , 1 ) Y\sim U(0,1) YU(0,1), 且它们独立.
    (1) 求联合密度 f ( x , y ) f(x,y) f(x,y);
    (2) 求 Z = X + Y Z=X+Y Z=X+Y 的密度函数;
    (3) 求 t 2 + 2 X t + Y = 0 t^2+2Xt+Y=0 t2+2Xt+Y=0 有实根的概率, 保留 3 位小数.

Solution: (1) 根据独立性, 有
f ( x , y ) = 1 2 e − 1 2 x , x > 0 , 0 < 1 < y . f(x,y)=\frac{1}{2}e^{-\frac{1}{2}x},\quad x>0,\quad 0<1<y. f(x,y)=21e21x,x>0,0<1<y.

(2) 作变量变换, 有
{ Z = X + Y , W = Y , ⇒ { z = x + y , w = y , ⇒ { x = z − w , y = w , ⇒ J = ∣ 1 − 1 0 1 ∣ = 1 , \begin{cases} Z=X+Y,\\ W=Y,\\ \end{cases}\Rightarrow \begin{cases} z=x+y,\\ w=y,\\ \end{cases}\Rightarrow \begin{cases} x=z-w,\\ y=w,\\ \end{cases}\Rightarrow J=\left| \begin{matrix} 1& -1\\ 0& 1\\ \end{matrix} \right|=1, {Z=X+Y,W=Y,{z=x+y,w=y,{x=zw,y=w,J= 1011 =1,
因此有
f Z , W ( z , w ) = f ( z − w , w ) = 1 2 e − z 2 + w 2 , z > w , 0 < w < 1 , f_{Z,W}\left( z,w \right) =f\left( z-w,w \right) =\frac{1}{2}e^{-\frac{z}{2}+\frac{w}{2}},\quad z>w,0<w<1, fZ,W(z,w)=f(zw,w)=21e2z+2w,z>w,0<w<1,
积掉 W W W, 得
f Z ( z ) = 1 2 e − z 2 ∫ 0 min ⁡ { z , 1 } e w 2 d w = e − z 2 ( e min ⁡ { z , 1 } 2 − 1 ) = { e − z 2 ( e 1 2 − 1 ) , z > 1 , 1 − e − z 2 , 0 < z < 1. f_Z\left( z \right) =\frac{1}{2}e^{-\frac{z}{2}}\int_0^{\min \left\{ z,1 \right\}}{e^{\frac{w}{2}}dw}=e^{-\frac{z}{2}}\left( e^{\frac{\min \left\{ z,1 \right\}}{2}}-1 \right) =\begin{cases} e^{-\frac{z}{2}}\left( e^{\frac{1}{2}}-1 \right) ,& z>1,\\ 1-e^{-\frac{z}{2}},& 0<z<1.\\ \end{cases} fZ(z)=21e2z0min{z,1}e2wdw=e2z(e2min{z,1}1)={e2z(e211),1e2z,z>1,0<z<1.

(3) Δ = 4 X 2 − 4 Y \Delta = 4X^2 -4Y Δ=4X24Y, 故所求概率为 P ( X 2 ≥ Y ) P(X^2\ge Y) P(X2Y), 有
P ( X 2 ≥ Y ) = ∫ 0 1 P ( X ≥ y ) f Y ( y ) d y = ∫ 0 1 e − y 2 d y = 8 ( 1 − 3 2 e − 1 2 ) = 0.721632. P\left( X^2\ge Y \right) =\int_0^1{P\left( X\ge \sqrt{y} \right) f_Y\left( y \right) dy}=\int_0^1{e^{-\frac{\sqrt{y}}{2}}dy}=8\left( 1-\frac{3}{2}e^{-\frac{1}{2}} \right) =0.721632. P(X2Y)=01P(Xy )fY(y)dy=01e2y dy=8(123e21)=0.721632.

  1. (10分) 假设检验问题: 给出两组正态总体数据 X , Y X,Y X,Y.
    (1) 检验 H 0 : σ 1 2 = σ 2 2 H_0:\sigma_1^2 =\sigma_2^2 H0:σ12=σ22;
    (2) 检验 H 0 : μ 1 = μ 2 H_0:\mu_1=\mu_2 H0:μ1=μ2.
  1. (25分) 有来自总体 f ( x , a ) = 2 x a 2 , 0 < x < a f(x,a)=\frac{2x}{a^2},0<x<a f(x,a)=a22x,0<x<a 的 i.i.d. 样本 x 1 , ⋯   , x n x_1,\cdots,x_n x1,,xn, 已知 a > 1 a>1 a>1.
    (1) 求 a a a 的矩估计 a ^ 1 \hat{a}_1 a^1, 最大似然估计 a ^ 2 \hat{a}_2 a^2, 以及 P ( 0 < X < a ) P(0<X<\sqrt{a}) P(0<X<a ) 的MLE;
    (2) a ^ 1 \hat{a}_1 a^1, a ^ 2 \hat{a}_2 a^2 是否为无偏估计, 若不是请修正;
    (3) 求 n ( a − a ^ 2 ) n(a-\hat{a}_2) n(aa^2) n → ∞ n\to \infty n 的渐近分布.

Solution: (1) 求总体期望 E ( X ) E(X) E(X), 利用 X a ∼ B e t a ( 2 , 1 ) \frac{X}{a}\sim Beta(2,1) aXBeta(2,1) 或直接积分有 E ( X ) = 2 3 a E(X)=\frac{2}{3}a E(X)=32a, 由替换原理, 得 a ^ 1 = 3 2 x ˉ \hat{a}_1=\frac{3}{2}\bar{x} a^1=23xˉ.

再写似然函数, 有
L ( a ) = 2 n ∏ i = 1 n x i a 2 n , a > max ⁡ { x ( n ) , 1 } , L\left( a \right) =\frac{2^n\prod_{i=1}^n{x_i}}{a^{2n}},\quad a>\max \left\{ x_{\left( n \right)},1 \right\} , L(a)=a2n2ni=1nxi,a>max{x(n),1},
可以看出似然函数关于 a a a 递减, 故有
a ^ 2 = max ⁡ { x ( n ) , 1 } = { 1 , x ( n ) < 1 , x ( n ) , x ( n ) ≥ 1. \hat{a}_2=\max \{x_{(n)},1\}=\begin{cases} 1,& x_{\left( n \right)}<1,\\ x_{\left( n \right)},& x_{\left( n \right)}\ge 1.\\ \end{cases} a^2=max{x(n),1}={1,x(n),x(n)<1,x(n)1.

(2) 由于 E ( x ˉ ) = 2 3 a E(\bar{x})=\frac{2}{3}a E(xˉ)=32a, 显然 a ^ 1 \hat{a}_1 a^1 无偏.

对于 a ^ 2 \hat{a}_2 a^2, 先求 x ( n ) x_{(n)} x(n) 的分布, 有
P ( x ( n ) ≤ t ) = P n ( X ≤ t ) = ( t a ) 2 n , f n ( t ) = 2 n t 2 n − 1 a 2 n , 0 < t < a , P\left( x_{\left( n \right)}\le t \right) =P^n\left( X\le t \right) =\left( \frac{t}{a} \right) ^{2n},\quad f_n\left( t \right) =\frac{2nt^{2n-1}}{a^{2n}},\quad 0<t<a, P(x(n)t)=Pn(Xt)=(at)2n,fn(t)=a2n2nt2n1,0<t<a,
实际上即为 x ( n ) a ∼ B e ( 2 n , 1 ) \frac{x_{(n)}}{a}\sim Be(2n,1) ax(n)Be(2n,1), 故有
E ( a ^ 2 ) = ∫ 0 1 f n ( t ) d t + ∫ 1 a t f n ( t ) d t = 1 a 2 n + 2 n 2 n + 1 ( a − 1 a 2 n ) = 2 n 2 n + 1 ⋅ a + 1 2 n + 1 ⋅ 1 a 2 n , E\left( \hat{a}_2 \right) =\int_0^1{f_n\left( t \right) dt}+\int_1^a{tf_n\left( t \right) dt}=\frac{1}{a^{2n}}+\frac{2n}{2n+1}\left( a-\frac{1}{a^{2n}} \right) =\frac{2n}{2n+1}\cdot a+\frac{1}{2n+1}\cdot \frac{1}{a^{2n}}, E(a^2)=01fn(t)dt+1atfn(t)dt=a2n1+2n+12n(aa2n1)=2n+12na+2n+11a2n1,
由于 a > 1 a>1 a>1, 故 1 a 2 n < 1 \frac{1}{a^{2n}}<1 a2n1<1, 因此
2 n 2 n + 1 ⋅ a + 1 2 n + 1 ⋅ 1 a 2 n < 2 n 2 n + 1 ⋅ a + 1 2 n + 1 < a . \frac{2n}{2n+1}\cdot a+\frac{1}{2n+1}\cdot \frac{1}{a^{2n}}<\frac{2n}{2n+1}\cdot a+\frac{1}{2n+1}<a. 2n+12na+2n+11a2n1<2n+12na+2n+11<a.
a ^ 2 \hat{a}_2 a^2 不无偏. 直接乘一个不含 a a a 的数不可能修正为无偏估计, 但我们发现在求期望的过程中, 如果写成
∫ 0 1 f n ( t ) d t + 2 n + 1 2 n ∫ 1 a t f n ( t ) d t = 1 a 2 n + ( a − 1 a 2 n ) = a , \int_0^1{f_n\left( t \right) dt}+\frac{2n+1}{2n}\int_1^a{tf_n\left( t \right) dt}=\frac{1}{a^{2n}}+\left( a-\frac{1}{a^{2n}} \right) =a, 01fn(t)dt+2n2n+11atfn(t)dt=a2n1+(aa2n1)=a,
则恰好是无偏估计, 这对应的估计量是
a ~ 2 = { 1 , x ( n ) < 1 , 2 n + 1 2 n x ( n ) , x ( n ) ≥ 1. \tilde{a}_2=\begin{cases} 1,& x_{\left( n \right)}<1,\\ \frac{2n+1}{2n}x_{\left( n \right)},& x_{\left( n \right)}\ge 1.\\ \end{cases} a~2={1,2n2n+1x(n),x(n)<1,x(n)1.

(3) 记 T n = n ( a − a ^ 2 ) T_n = n(a-\hat{a}_2) Tn=n(aa^2), 则有
P ( T n ≤ t ) = P ( n ( a − a ^ 2 ) ≤ t ) = P ( a − a ^ 2 ≤ t n ) = P ( a ^ 2 ≥ a − t n ) , P\left( T_n\le t \right) =P\left( n\left( a-\hat{a}_2 \right) \le t \right) =P\left( a-\hat{a}_2\le \frac{t}{n} \right) =P\left( \hat{a}_2\ge a-\frac{t}{n} \right) , P(Tnt)=P(n(aa^2)t)=P(aa^2nt)=P(a^2ant),
对于 t > 0 t>0 t>0, 总有 n n n 足够大使得 a − t n > 1 a-\frac{t}{n}>1 ant>1, 因此
P ( a ^ 2 ≥ a − t n ) = P ( x ( n ) ≥ a − t n ) = 1 − ( 1 − t a n ) 2 n → 1 − e − 2 t a , t > 0 , P\left( \hat{a}_2\ge a-\frac{t}{n} \right) =P\left( x_{\left( n \right)}\ge a-\frac{t}{n} \right) =1-\left( 1-\frac{t}{an} \right) ^{2n}\rightarrow 1-e^{-\frac{2t}{a}},\quad t>0, P(a^2ant)=P(x(n)ant)=1(1ant)2n1ea2t,t>0,
这说明 n ( a − a ^ 2 ) → d E x p ( 2 a ) n(a-\hat{a}_2)\xrightarrow{d}Exp(\frac{2}{a}) n(aa^2)d Exp(a2).

  1. (15分) 叙述题:(1) 叙述多重共线性的定义;
    (2) 如何判断多重共线性:
    (3) 如何消除多重共线性:
    (4) 叙述自变量的选择标准.

Solution: (1) 在回归分析中,如果两个或两个以上自变量之间存在相关性,这种自变量之间的相关性,就称作多重共线性,也称作自变量间的相关性。多重共线性的存在违背了线性回归模型的基本假设,变量之间的线性相关性将会导致矩阵 X T X X^TX XTX 不满秩,进而导致最小二乘估计不唯一。

(2) 可以借助方差膨胀因子 VIF 来判断共线性,计算公式是
V I F j = 1 1 − R j 2 , VIF_j = \frac{1}{1-R_j^2}, VIFj=1Rj21,
一般我们认为 VIF > 10 时,存在多重共线性,该特征需要删除。

我们也可以分析矩阵 X T X X^TX XTX 的特征值,如果该矩阵的最小特征值非常接近于 0,我们也认为存在多重共线性。

(3) 可利用逐步回归筛选并剔除引起多重共线性的变量,其具体步骤如下:先用被解释变量对每一个所考虑的解释变量做简单回归,然后以对被解释变量贡献最大的解释变量所对应的回归方程为基础,再逐步引入其余解释变量。经过逐步回归,使得最后保留在模型中的解释变量既是重要的,又没有严重多重共线性。

(4) 在模型中加入自变量时,要尽量使得:残差平方和缩小或决定系数增大,若某一自变量被引入模型后 SSE 减小很多,说明该变量对反映变量 y y y 的作用大,可被引入;反之,说明其对 y y y 的作用小,不应该被引入。此外,还可以根据赤池信息准则(AIC)、贝叶斯信息准则(BIC)、对数似然函数值(LLH)等方法判断。

  1. (25分) 设有来自 f ( x ; λ ) = λ e − λ x f(x;\lambda)=\lambda e^{-\lambda x} f(x;λ)=λeλx 指数分布的 i.i.d. 样本 x 1 , ⋯   , x n x_1,\cdots,x_n x1,,xn, 但由于某种原因只能观测到 A i = I { a i < x i < b i } A_i=I_{\{a_i<x_i<b_i\}} Ai=I{ai<xi<bi}, 其中 a i , b i a_i,b_i ai,bi 是给定常数, i = 1 , 2 , ⋯   , n i=1,2,\cdots,n i=1,2,,n.
    (1) 写出 ( A 1 , ⋯   , A n ) (A_1,\cdots,A_n) (A1,,An) 对应的对数似然函数 ℓ A ( λ ) \ell_A(\lambda) A(λ), 同时写出完整样本 ( x 1 , ⋯   , x n ) (x_1,\cdots,x_n) (x1,,xn) 对应的对数似然函数 ℓ X ( λ ) \ell_X(\lambda) X(λ);
    (2) 写出基于 ℓ A ( λ ) \ell_A(\lambda) A(λ) 所求 MLE 满足的等式;
    (3) 分别考虑两个步骤:
    (i) E 步: 考虑 X ∼ E x p ( λ k ) X\sim Exp(\lambda_k) XExp(λk), 求条件期望
    Q ( λ ∣ λ k ) = E [ ℓ X ( λ ) ∣ A , λ k ] , Q(\lambda|\lambda_k) = E[\ell_X(\lambda)|A,\lambda_k], Q(λλk)=E[X(λ)A,λk],
    (ii) M 步: 极大化 Q ( λ ) Q(\lambda) Q(λ), 即
    λ k + 1 = a r g m a x λ Q ( λ ∣ λ k ) . \lambda_{k+1} =\underset{\lambda}{\mathrm{argmax}} Q(\lambda|\lambda_k). λk+1=λargmaxQ(λλk).
    (4) 证明: 通过两个步骤迭代得到的序列 λ n → λ ^ \lambda_n \to \hat{\lambda} λnλ^, 其中 λ ^ \hat{\lambda} λ^ 是基于 ℓ A ( λ ) \ell_A(\lambda) A(λ) 求得的 MLE. (提示:和 a i , b i , λ k , λ 0 a_i,b_i,\lambda_k,\lambda_0 ai,bi,λk,λ0 有关)

Solution: (1) 每个 A i A_i Ai 都是两点分布, 其参数是
p i ( λ ) = P ( a i < x i < b i ) = e − λ a i − e − λ b i , p_i(\lambda) = P(a_i<x_i<b_i)= e^{-\lambda a_i}-e^{-\lambda b_i}, pi(λ)=P(ai<xi<bi)=eλaieλbi,
因此有
L A ( λ ) = ∏ i = 1 n p i A i ( 1 − p i ) 1 − A i = ∏ i = 1 n p i A i ⋅ ∏ i = 1 n ( 1 − p i ) 1 − A i , L_A\left( \lambda \right) =\prod_{i=1}^n{p_{i}^{A_i}\left( 1-p_i \right) ^{1-A_i}}=\prod_{i=1}^n{p_{i}^{A_i}}\cdot \prod_{i=1}^n{\left( 1-p_i \right) ^{1-A_i}}, LA(λ)=i=1npiAi(1pi)1Ai=i=1npiAii=1n(1pi)1Ai,
故有
ℓ A ( λ ) = ∑ i = 1 n A i ln ⁡ p i + ∑ i = 1 n ( 1 − A i ) ln ⁡ ( 1 − p i ) . \ell _A\left( \lambda \right) =\sum_{i=1}^n{A_i\ln p_i}+\sum_{i=1}^n{\left( 1-A_i \right) \ln \left( 1-p_i \right)}. A(λ)=i=1nAilnpi+i=1n(1Ai)ln(1pi).

而全样本对应的对数似然函数是指数分布的联合密度取对数, 即
ℓ X ( λ ) = n ln ⁡ λ − λ ∑ i = 1 n x i . \ell _X\left( \lambda \right) =n\ln \lambda -\lambda \sum_{i=1}^n{x_i}. X(λ)=nlnλλi=1nxi.

(2) 记 q i ( λ ) = a i e − λ a i − b i e − λ b i q_i(\lambda) = a_ie^{-\lambda a_i} - b_ie^{-\lambda b_i} qi(λ)=aieλaibieλbi, 实际上即 q i = − ∂ p i ∂ λ q_i = -\frac{\partial p_i}{\partial \lambda} qi=λpi, 求导有
∂ ℓ A ∂ λ = ∑ i = 1 n A i − q i p i + ∑ i = 1 n ( 1 − A i ) q i 1 − p i = − ∑ i = 1 n q i ( A i p i − 1 − A i 1 − p i ) = − ∑ i = 1 n q i A i − p i p i ( 1 − p i ) , \begin{aligned} \frac{\partial \ell _A}{\partial \lambda}&=\sum_{i=1}^n{A_i\frac{-q_i}{p_i}}+\sum_{i=1}^n{\left( 1-A_i \right) \frac{q_i}{1-p_i}}\\ &=-\sum_{i=1}^n{q_i\left( \frac{A_i}{p_i}-\frac{1-A_i}{1-p_i} \right)}=-\sum_{i=1}^n{q_i\frac{A_i-p_i}{p_i\left( 1-p_i \right)},}\\ \end{aligned} λA=i=1nAipiqi+i=1n(1Ai)1piqi=i=1nqi(piAi1pi1Ai)=i=1nqipi(1pi)Aipi,
因此 MLE λ ^ \hat{\lambda} λ^ 满足
∑ i = 1 n q i ( λ ^ ) A i − p i ( λ ^ ) p i ( λ ^ ) ( 1 − p i ( λ ^ ) ) = 0. \sum_{i=1}^n{q_i\left( \hat{\lambda} \right) \frac{A_i-p_i\left( \hat{\lambda} \right)}{p_i\left( \hat{\lambda} \right) \left( 1-p_i\left( \hat{\lambda} \right) \right)}}=0. i=1nqi(λ^)pi(λ^)(1pi(λ^))Aipi(λ^)=0.

(3) 先求 E [ x k ∣ A k ] E[x_k|A_k] E[xkAk], 有
E [ x i ∣ A i = 1 ] = E [ x i I { a i < x i < b i } ] P ( a i < x i < b i ) = 1 λ + q i p i , E\left[ x_i\mid A_i=1 \right] =\frac{E\left[ x_iI_{\left\{ a_i<x_i<b_i \right\}} \right]}{P\left( a_i<x_i<b_i \right)}=\frac{1}{\lambda}+\frac{q_i}{p_i}, E[xiAi=1]=P(ai<xi<bi)E[xiI{ai<xi<bi}]=λ1+piqi,
其中分子利用了
∫ a i b i λ x e − λ x d x = 1 λ ∫ λ a i λ b i u e − u d u = 1 λ [ ( λ a i + 1 ) e − λ a i − ( λ b i + 1 ) e − λ b i ] = q i + p i λ . \int_{a_i}^{b_i}{\lambda xe^{-\lambda x}dx}=\frac{1}{\lambda}\int_{\lambda a_i}^{\lambda b_i}{ue^{-u}du}=\frac{1}{\lambda}\left[ \left( \lambda a_i+1 \right) e^{-\lambda a_i}-\left( \lambda b_i+1 \right) e^{-\lambda b_i} \right] =q_i+\frac{p_i}{\lambda}. aibiλxeλxdx=λ1λaiλbiueudu=λ1[(λai+1)eλai(λbi+1)eλbi]=qi+λpi.
同理用 E [ x i I { x ∉ ( a i , b i ) } ] = E [ x i ] − E [ x i I { x i ∈ ( a i , b i ) } ] E[x_iI_{\{x\notin (a_i,b_i)\}}]=E[x_i]-E[x_iI_{\{x_i\in(a_i,b_i)\}}] E[xiI{x/(ai,bi)}]=E[xi]E[xiI{xi(ai,bi)}], 有
E [ x i ∣ A i = 0 ] = 1 − p i λ − q i 1 − p i = 1 λ − q i 1 − p i . E\left[ x_i\mid A_i=0 \right] =\frac{\frac{1-p_i}{\lambda}-q_i}{1-p_i}=\frac{1}{\lambda}-\frac{q_i}{1-p_i}. E[xiAi=0]=1piλ1piqi=λ11piqi.

因此有 E 步是:
Q ( λ ∣ λ k ) = E [ ℓ X ( λ ) ∣ A , λ k ] = n ln ⁡ λ − λ ∑ i = 1 n E [ x i ∣ A i , λ k ] = n ln ⁡ λ − λ ∑ i = 1 n ( 1 λ k + q i ( λ k ) ( A i p i ( λ k ) − 1 − A i 1 − p i ( λ k ) ) ) = n ln ⁡ λ − n λ λ k − λ ∑ i = 1 n q i ( λ k ) A i − p i ( λ k ) p i ( λ k ) ( 1 − p i ( λ k ) ) . \begin{aligned} Q\left( \lambda |\lambda _k \right) &=E\left[ \ell _X\left( \lambda \right) |A,\lambda _k \right]\\ &=n\ln \lambda -\lambda \sum_{i=1}^n{E\left[ x_i\mid A_i,\lambda _k \right]}\\ &=n\ln \lambda -\lambda \sum_{i=1}^n{\left( \frac{1}{\lambda _k}+q_i\left( \lambda _k \right) \left( \frac{A_i}{p_i\left( \lambda _k \right)}-\frac{1-A_i}{1-p_i\left( \lambda _k \right)} \right) \right)}\\ &=n\ln \lambda -\frac{n\lambda}{\lambda _k}-\lambda \sum_{i=1}^n{q_i\left( \lambda _k \right)}\frac{A_i-p_i\left( \lambda _k \right)}{p_i\left( \lambda _k \right) \left( 1-p_i\left( \lambda _k \right) \right)}.\\ \end{aligned} Q(λλk)=E[X(λ)A,λk]=nlnλλi=1nE[xiAi,λk]=nlnλλi=1n(λk1+qi(λk)(pi(λk)Ai1pi(λk)1Ai))=nlnλλkλi=1nqi(λk)pi(λk)(1pi(λk))Aipi(λk).

再考虑 M 步: 对 Q ( λ ∣ λ k ) Q(\lambda|\lambda_k) Q(λλk) 求极大化(注意 λ k \lambda_k λk 是常数, 只有 λ \lambda λ 是变量), 可以求导得
∂ Q ∂ λ = n λ − n λ k − ∑ i = 1 n q i ( λ k ) A i − p i ( λ k ) p i ( λ k ) ( 1 − p i ( λ k ) ) , \frac{\partial Q}{\partial \lambda}=\frac{n}{\lambda}-\frac{n}{\lambda _k}-\sum_{i=1}^n{q_i\left( \lambda _k \right)}\frac{A_i-p_i\left( \lambda _k \right)}{p_i\left( \lambda _k \right) \left( 1-p_i\left( \lambda _k \right) \right)}, λQ=λnλkni=1nqi(λk)pi(λk)(1pi(λk))Aipi(λk),
解得极值点满足
1 λ = 1 λ k + 1 n ∑ i = 1 n q i ( λ k ) A i − p i ( λ k ) p i ( λ k ) ( 1 − p i ( λ k ) ) , \frac{1}{\lambda}=\frac{1}{\lambda _k}+\frac{1}{n}\sum_{i=1}^n{q_i\left( \lambda _k \right)}\frac{A_i-p_i\left( \lambda _k \right)}{p_i\left( \lambda _k \right) \left( 1-p_i\left( \lambda _k \right) \right)}, λ1=λk1+n1i=1nqi(λk)pi(λk)(1pi(λk))Aipi(λk),
故有
λ k + 1 = 1 1 λ k + 1 n ∑ i = 1 n q i ( λ k ) A i − p i ( λ k ) p i ( λ k ) ( 1 − p i ( λ k ) ) . \lambda _{k+1}=\frac{1}{\frac{1}{\lambda _k}+\frac{1}{n}\sum_{i=1}^n{q_i\left( \lambda _k \right)}\frac{A_i-p_i\left( \lambda _k \right)}{p_i\left( \lambda _k \right) \left( 1-p_i\left( \lambda _k \right) \right)}}. λk+1=λk1+n1i=1nqi(λk)pi(λk)(1pi(λk))Aipi(λk)1.

(4) 该序列满足
1 λ k + 1 = 1 λ k + 1 n ∑ i = 1 n q i ( λ k ) A i − p i ( λ k ) p i ( λ k ) ( 1 − p i ( λ k ) ) , \frac{1}{\lambda _{k+1}}= \frac{1}{\lambda_k} + \frac{1}{n}\sum_{i=1}^n{q_i\left( \lambda _k \right)}\frac{A_i-p_i\left( \lambda _k \right)}{p_i\left( \lambda _k \right) \left( 1-p_i\left( \lambda _k \right) \right)}, λk+11=λk1+n1i=1nqi(λk)pi(λk)(1pi(λk))Aipi(λk),
D ( λ k ) = − ∑ i = 1 n q i ( λ k ) A i − p i ( λ k ) p i ( λ k ) ( 1 − p i ( λ k ) ) D(\lambda_k) =- \sum_{i=1}^n{q_i\left( \lambda _k \right)}\frac{A_i-p_i\left( \lambda _k \right)}{p_i\left( \lambda _k \right) \left( 1-p_i\left( \lambda _k \right) \right)} D(λk)=i=1nqi(λk)pi(λk)(1pi(λk))Aipi(λk), 这恰好是 ℓ A \ell_A A λ k \lambda_k λk 点的导数, 而
1 λ k + 1 = 1 λ k − 1 n ⋅ D ( λ k ) , \frac{1}{\lambda_{k+1}} = \frac{1}{\lambda_k} - \frac{1}{n} \cdot D(\lambda_k), λk+11=λk1n1D(λk),
该序列保证了 λ k \lambda_k λk 在导数的同方向迭代, 即保证了函数值 ℓ A \ell_A A 的上升, 因此 { λ n } \{\lambda_n\} {λn} 一定收敛到 ℓ A \ell_A A 的某个驻点, 即导数为 0 的点, 即 λ ^ \hat{\lambda} λ^.

  • 0
    点赞
  • 0
    收藏
    觉得还不错? 一键收藏
  • 0
    评论
评论
添加红包

请填写红包祝福语或标题

红包个数最小为10个

红包金额最低5元

当前余额3.43前往充值 >
需支付:10.00
成就一亿技术人!
领取后你会自动成为博主和红包主的粉丝 规则
hope_wisdom
发出的红包
实付
使用余额支付
点击重新获取
扫码支付
钱包余额 0

抵扣说明:

1.余额是钱包充值的虚拟货币,按照1:1的比例进行支付金额的抵扣。
2.余额无法直接购买下载,可以购买VIP、付费专栏及课程。

余额充值